OLYMPIADE DE MATHÉMATIQUES : 2âŋ - 2áĩ = 4080

āđāļŠāļĢāđŒ
āļāļąāļ‡
  • āđ€āļœāļĒāđāļžāļĢāđˆāđ€āļĄāļ·āđˆāļ­ 7 āļ˜.āļ„. 2022
  • ðŸŽŊ Muscle ton cerveau en faisant de ton quotidien un exercice de maths que tu sauras rÃĐsoudre 💊 : hedacademy.fr
    Question tirÃĐe des olympiades de maths.
    DÃĐterminer les entiers positifs n et m tels que
    2âŋ - 2áĩ = 4080

āļ„āļ§āļēāļĄāļ„āļīāļ”āđ€āļŦāđ‡āļ™ • 314

  • @draconsilver
    @draconsilver āļ›āļĩāļ—āļĩāđˆāđāļĨāđ‰āļ§ +184

    Je suis embÊtÃĐ parce que je savais que 4096 ÃĐtait une puissance de 2. Du coup 16 aussi et du coup sans raisonner j’avais les bonnes puissances ^^

    • @alphacentauri7757
      @alphacentauri7757 āļ›āļĩāļ—āļĩāđˆāđāļĨāđ‰āļ§ +6

      Pareil pour moi

    • @samueljehanno
      @samueljehanno āļ›āļĩāļ—āļĩāđˆāđāļĨāđ‰āļ§ +3

      Pareil

    • @Laggron93
      @Laggron93 āļ›āļĩāļ—āļĩāđˆāđāļĨāđ‰āļ§ +36

      On est beaucoup dans ce cas, la difficultÃĐ ÃĐtait de prouver l'unicitÃĐ de la solution!

    • @Axel_Roffi
      @Axel_Roffi āļ›āļĩāļ—āļĩāđˆāđāļĨāđ‰āļ§ +2

      idem pour moi, 4096=4080+ 2^4...et on avait 2^n-2^m+2^4=4096 soit 2^n-2^m=2^12-2^4 donc on avait bien 12 et 4 mais je me demandais si c'ÃĐtait les seules solutions possibles pour n et m

    • @florianbasier
      @florianbasier āļ›āļĩāļ—āļĩāđˆāđāļĨāđ‰āļ§ +3

      @@Axel_Roffi Pareil. C'est dommage d'ailleurs que @hedacademy ne le mentionne pas. Quand on arrive à 2^m * Ximpair = 4080, la decomposition en facteurs premiers ÃĐtant unique on a donc 2^m=2^3 et (2^k-1)=3*5*17, ce qui prouve l'unicitÃĐ je suis d'accord avec toi

  • @abdelghanibenabbou2990
    @abdelghanibenabbou2990 āļ›āļĩāļ—āļĩāđˆāđāļĨāđ‰āļ§ +13

    Vous Êtes vraiment formidable....
    On aime votre façon de traiter les problÃĻmes.

  • @NOAR-uw2jl
    @NOAR-uw2jl āļ›āļĩāļ—āļĩāđˆāđāļĨāđ‰āļ§ +11

    C'est beau! 😘
    Je retiens la dÃĐmonstration. Pas mÃĐcontent, je n'ai pas su aller au delà de la formule factorisÃĐe avec des puissances inconnues mais le dÃĐbut ÃĐtait bon, restait la logique qui opÃĐrait aprÃĻs ☚ïļ

  • @Carlos31416
    @Carlos31416 āļ›āļĩāļ—āļĩāđˆāđāļĨāđ‰āļ§ +1

    Je trouve ce mode de rÃĐsolution trÃĻs intÃĐressant et ÃĐlÃĐgant. En parlant d'autres chaÃŪnes, la chaÃŪne ÂŦMultiply divideÂŧ a d'autres exercices de ce type, un peu plus relevÃĐs, soit en base 2 ou en base 3, ce qui complique les choses, car si vous avez fait de l'informatique les bases 2 ça va tout seul, alors que la base 3...

  • @guypannetier7424
    @guypannetier7424 āļ›āļĩāļ—āļĩāđˆāđāļĨāđ‰āļ§ +4

    Magnifique.... Un rÃĐgal à voir.

  • @jeffh.8251
    @jeffh.8251 āļ›āļĩāļ—āļĩāđˆāđāļĨāđ‰āļ§ +7

    super exercice, jour aprÃĻs jour on s'amÃĐliore, et on dit MERCI QUI ???? MERCI HEDACADEMY's brothers

  • @midaway_-
    @midaway_- āļ›āļĩāļ—āļĩāđˆāđāļĨāđ‰āļ§ +1

    C’est passionnant moi j’avais pas du tout fait comme sa j’avais fait des test avec 4096 mais maintenant que j’ai vu le raisonnement je rÃĐalise que je ne m’y suis pas pris de la bonne maniÃĻre pour trouver la solution merci

  • @laurentsoria1528
    @laurentsoria1528 āļ›āļĩāļ—āļĩāđˆāđāļĨāđ‰āļ§

    TrÃĐs joli, et trÃĐs stimulant .
    Bravo

  • @Truuffe
    @Truuffe āļ›āļĩāļ—āļĩāđˆāđāļĨāđ‰āļ§ +5

    TrÃĻs intÃĐressant 💊💊

  • @alestane2
    @alestane2 āļ›āļĩāļ—āļĩāđˆāđāļĨāđ‰āļ§ +7

    Je vois beaucoup de gens qui disent avoir trouvÃĐ les valeurs parce qu'ils ont reconnu 4096 ou autre. C'est bien, vous avez trouvÃĐ une solution. Selon la façon dont le problÃĻme ÃĐtait posÃĐ, ça pouvait suffire ou pas. En tous cas, sans la dÃĐmonstration, vous pouvez dire que vous avez *une* solution mais vous ne pouvez pas dire que c'est la seule solution possible.

  • @sonia.bouzouita
    @sonia.bouzouita āļ›āļĩāļ—āļĩāđˆāđāļĨāđ‰āļ§ +1

    Trop cool ! J'adore ðŸĪĐ

  • @gragsDD5654
    @gragsDD5654 āļ›āļĩāļ—āļĩāđˆāđāļĨāđ‰āļ§ +1

    Une autre façon de trouver la solution tout en prouvant l'unicitÃĐ de celle-ci:
    On constate que n >= 12 car il faut que 2^n > 4080
    Maintenant en prenant n > 12 on peut essayer de minimiser 2^n -2^m pour ça on choisi m=n-1 on a alors:
    2^n - 2^(n-1) = 2^(n-1)
    Or cette fonction est strictement croissante et son premier terme (pour n=13) est 4096 > 4080
    Donc pour tout n > 12, quelque soit m < n on a pas de solution.
    Il reste uniquement n = 12 et on dÃĐduit m = 4

    • @cathycai
      @cathycai āļŦāļĨāļēāļĒāđ€āļ”āļ·āļ­āļ™āļāđˆāļ­āļ™

      Oui plus rapide et plus fin comme raisonnement

  • @raphaelsbr1899
    @raphaelsbr1899 āļ›āļĩāļ—āļĩāđˆāđāļĨāđ‰āļ§ +1

    TrÃĻs propre merci !

  • @grouchomarx6156
    @grouchomarx6156 āļ›āļĩāļ—āļĩāđˆāđāļĨāđ‰āļ§

    J'adore ce type!

  • @pierrettebalazut9407
    @pierrettebalazut9407 āļ›āļĩāļ—āļĩāđˆāđāļĨāđ‰āļ§

    Oui ça m'a plut et en plus j'ai compris.
    Merci

  • @pascalgallet5931
    @pascalgallet5931 āļ›āļĩāļ—āļĩāđˆāđāļĨāđ‰āļ§ +10

    Super intÃĐressant Professeur. AmitiÃĐs.

  • @fanderyosensei9937
    @fanderyosensei9937 5 āļŦāļĨāļēāļĒāđ€āļ”āļ·āļ­āļ™āļāđˆāļ­āļ™

    Salut, j'adore vos exercices

  • @user-cp5fx9rg2f
    @user-cp5fx9rg2f 5 āļŦāļĨāļēāļĒāđ€āļ”āļ·āļ­āļ™āļāđˆāļ­āļ™

    Merci pour l'explication😊

  • @cyruschang1904
    @cyruschang1904 āļ›āļĩāļ—āļĩāđˆāđāļĨāđ‰āļ§ +3

    2^n - 2^m = 4080 = 8 x 510 = 8 x 2 x 255 = 2^4 x 255
    (2^n - 2^m)/2^4 = 255
    2^(n - 4) - 2^(m - 4) = 255
    On sait que 2^8 = 256
    Donc 2^(n - 4) - 2^(m - 4) = 255 = 2^8 - 1
    n - 4 = 8 et m - 4 = 0 => n = 12 et m = 4

  • @user-ue1wi5cd7u
    @user-ue1wi5cd7u 3 āļŦāļĨāļēāļĒāđ€āļ”āļ·āļ­āļ™āļāđˆāļ­āļ™

    Merci professeur

  • @JeanDeLaCroix_
    @JeanDeLaCroix_ āļ›āļĩāļ—āļĩāđˆāđāļĨāđ‰āļ§

    J'avais rÃĐussi en faisait exactement la mÊme chose (factoriser, changer de variable, dÃĐcomposer en facteurs premiers), mais j'ai pas pensÃĐ Ã  sÃĐparer pair/impaire, ça fait gagner un peu de temps.

  • @djalikaassiabassande9689
    @djalikaassiabassande9689 āļ›āļĩāļ—āļĩāđˆāđāļĨāđ‰āļ§

    Merci beaucoup

  • @fruixfraise7708
    @fruixfraise7708 āļ›āļĩāļ—āļĩāđˆāđāļĨāđ‰āļ§ +4

    Let's go j'ai mit un commentaire il y a quelques vidÃĐos pour des questions d'olympiades, je sais pas si c'est grÃĒce à ça mais merci ! J'adore la vidÃĐo

    • @hedacademy
      @hedacademy  āļ›āļĩāļ—āļĩāđˆāđāļĨāđ‰āļ§ +4

      Si si je l’avais vu, c’ÃĐtait un petit clin d’œil en plus en le citant. En vrai plusieurs vidÃĐos sont inspirÃĐes des olympiades, j’oublie de le mentionner. Mais cette fois-ci grÃĒce à toi j’y ai pensÃĐ ðŸ˜…ðŸ‘ðŸž

    • @fruixfraise7708
      @fruixfraise7708 āļ›āļĩāļ—āļĩāđˆāđāļĨāđ‰āļ§ +3

      Merci beaucoup ! On voit que tu lis les retours ça fait plaisir : )

  • @druzicka2010
    @druzicka2010 6 āļŦāļĨāļēāļĒāđ€āļ”āļ·āļ­āļ™āļāđˆāļ­āļ™

    bon exemple et l'approche pour le rÃĐsoudre. ça va sans doute m'ÃĐclairer sur un calcul de capacitÃĐ de disque dur...

  • @papiou3243
    @papiou3243 āļ›āļĩāļ—āļĩāđˆāđāļĨāđ‰āļ§ +1

    2^n-2^m = 4080
    2^n = 4080+2^m
    A partir de là il suffit de calculer toutes les puissances de 2 jusqu'à un nombre supÃĐrieur à 4080.
    Pour quelqu'un qui s'y connait un peu en informatique, il connait cela :
    On sait que 1024 est 2^10 (2, 4, 8, 16, 32, 64, 128, 256, 512, 1024)
    2048, 4096 (hop ça y est nous sommes pile au dessus de 4080)
    Or 4096 = 2^12 = 2^n
    n=12
    4096-4080 = 16 = 2^4 = 2^m
    m=4

  • @erramdaniabdellah2984
    @erramdaniabdellah2984 āļ›āļĩāļ—āļĩāđˆāđāļĨāđ‰āļ§ +2

    TrÃĻs astucieux. Chapeau!!

    • @LouisLeCrack
      @LouisLeCrack 5 āļŦāļĨāļēāļĒāđ€āļ”āļ·āļ­āļ™āļāđˆāļ­āļ™

      trÃĻs facile surtout oui... extrÊmement facile meme

  • @zzk501
    @zzk501 āļ›āļĩāļ—āļĩāđˆāđāļĨāđ‰āļ§

    Bsahtek khouya

  • @kitsune6834
    @kitsune6834 āļ›āļĩāļ—āļĩāđˆāđāļĨāđ‰āļ§ +6

    Yes enfin j'ai trouvÃĐ une bonne rÃĐponse avant la correction :)
    Bon j'avoue comme je suis en filiÃĻre numÃĐrique et ÃĐlectronique je connaissais mes puissances de 2.
    Merci pour ce partage, vous Êtes TOP

    • @42ArthurDent42
      @42ArthurDent42 āļ›āļĩāļ—āļĩāđˆāđāļĨāđ‰āļ§

      J'ai envie de parier que tu avais eu l'intuition d'une solution ÃĐvidente, mais que tu n'as pas cherchÃĐ Ã  prouver qu'il n'y en avait pas d'autres...

    • @kitsune6834
      @kitsune6834 āļ›āļĩāļ—āļĩāđˆāđāļĨāđ‰āļ§ +3

      @@42ArthurDent42 Bah en fait, jsuis ÃĐlectronicien donc j'ai l'habitude des mots binaires... Le 2^10 je connaissais dÃĐjà et donc en montant juste deux crans au dessus t'as 2^12 qui fait 4096 et donc pour soustraire 16 je sais que 2^4 = 16. Oui je me suis pas trop cassÃĐ la tÊte 😅😂

    • @42ArthurDent42
      @42ArthurDent42 āļ›āļĩāļ—āļĩāđˆāđāļĨāđ‰āļ§ +1

      @@kitsune6834 Oui, la solution (4,12) est ÃĐvidente ! l'intÃĐrÊt de l'exo est de montrer que c'est la seule, et là c'est plus dur !

    • @kitsune6834
      @kitsune6834 āļ›āļĩāļ—āļĩāđˆāđāļĨāđ‰āļ§ +1

      @@42ArthurDent42 Exactement si la question c'est montrer que... Jsuis foutu ðŸĨē mais grÃĒce à ce super prof je pourrais prÃĐtendre faire un autre de ce genre 👍.

    • @motivationallessons4life
      @motivationallessons4life 6 āļŦāļĨāļēāļĒāđ€āļ”āļ·āļ­āļ™āļāđˆāļ­āļ™

      @@42ArthurDent42tu sais comment on peut demontrer l’unicite

  • @antoinet1304
    @antoinet1304 āļ›āļĩāļ—āļĩāđˆāđāļĨāđ‰āļ§ +1

    ici un vieux (MPSI 96) j'adore ;) CONTINUE

  • @chihiroetledragonmythique9404
    @chihiroetledragonmythique9404 āļ›āļĩāļ—āļĩāđˆāđāļĨāđ‰āļ§ +1

    Excellent.

  • @dastat7443
    @dastat7443 āļ›āļĩāļ—āļĩāđˆāđāļĨāđ‰āļ§ +2

    1024, 2048, 4096, 8192, ...
    Quand on taf dans l'informatique on est habituÃĐ Ã  ces puissances là.
    Je l'ai fait de tÊte en moins de temps mais l'approche ÃĐquation est super intÃĐressante.
    Merci.

    • @hmz-25
      @hmz-25 āļ›āļĩāļ—āļĩāđˆāđāļĨāđ‰āļ§

      Hahahah pareil

    • @paolo_mrtt
      @paolo_mrtt 4 āļŦāļĨāļēāļĒāđ€āļ”āļ·āļ­āļ™āļāđˆāļ­āļ™ +1

      Dis toi que j'ai pensÃĐ exactement à ton commentaire dans un exercice d'olympiades de maths pour utiliser les nombres binaires, sans quoi j'aurais pas trouvÃĐ la solution. Merci ! x)

  • @wilsonlouis-jeune3288
    @wilsonlouis-jeune3288 āļ›āļĩāļ—āļĩāđˆāđāļĨāđ‰āļ§

    Merci

  • @fanderyosensei9937
    @fanderyosensei9937 5 āļŦāļĨāļēāļĒāđ€āļ”āļ·āļ­āļ™āļāđˆāļ­āļ™

    S'il vous plaÃŪt je voudrais que vous continuez de traiter les exos olympiade de maths

  • @matthieuappenzeller9244
    @matthieuappenzeller9244 āļ›āļĩāļ—āļĩāđˆāđāļĨāđ‰āļ§

    AprÃĻs n>m => n-1â‰Ĩm (car n et m entiers) => 2^(n-1)â‰Ĩ2^m => 2^n-2^mâ‰Ĩ2^n-2^(n-1)=2^(n-1).
    On a donc, s'il y a une solution, 2^nâ‰Ĩ2^n-2^m=4080â‰Ĩ2^(n-1). Donc n=12 puis m=4.

  • @karimasaadia7055
    @karimasaadia7055 āļ›āļĩāļ—āļĩāđˆāđāļĨāđ‰āļ§

    Je t'adore

  • @pascalvaccaro371
    @pascalvaccaro371 āļ›āļĩāļ—āļĩāđˆāđāļĨāđ‰āļ§ +2

    Étant informaticien, je connais trÃĻs bien mes puissances de 2 alors j'ai fait 4080 = 4096 - 16 = 2^12 - 2^4 mais j'ai bien aimÃĐ la solution factorisation !

    • @pierreg9898
      @pierreg9898 āļ›āļĩāļ—āļĩāđˆāđāļĨāđ‰āļ§ +4

      Oui c’est une solution triviale mais l’enjeu ÃĐtant ici de prouver aussi l’unicitÃĐ de la solution ce qui n’est pas le cas avec votre mÃĐthode

  • @vinceguemat3751
    @vinceguemat3751 āļ›āļĩāļ—āļĩāđˆāđāļĨāđ‰āļ§ +5

    sinon, tu ÃĐcrit en binaire le nombre 4080 = 1111 1111 0000 et donc pour obtenir ca a partir de 2 puissances de 2 il faut 1 0000 0000 0000 - 1 0000 soit 2^12 - 2^4

    • @MrChompenrage
      @MrChompenrage āļ›āļĩāļ—āļĩāđˆāđāļĨāđ‰āļ§

      Je pense qu'il manque un 1 dans ta dÃĐcomposition de 4080 et un 0 dans celle de 2^12

    • @vinceguemat3751
      @vinceguemat3751 āļ›āļĩāļ—āļĩāđˆāđāļĨāđ‰āļ§

      @@MrChompenrage oui, probable

  • @cofbmaitres1177
    @cofbmaitres1177 āļ›āļĩāļ—āļĩāđˆāđāļĨāđ‰āļ§ +3

    En partant de 2^10=1024, on trouve que 2^12=2Âēx2^10=4x1024=4096. On trouve alors que 4080=4096-16=2^12-2^4. On peut alors conclure que n=12 et m=4. Voilà, je ne sais pas si c'est assez rigoureux pour un excercice oÃđ on doit rÃĐdiger, mais ça permet d'aller rapidement à la rÃĐponse. La seule limite c'est qu'on n'a pas dÃĐmontrÃĐ que ce sont les seules solutions

    • @42ArthurDent42
      @42ArthurDent42 āļ›āļĩāļ—āļĩāđˆāđāļĨāđ‰āļ§ +6

      oui ahah t'as rien prouvÃĐ du tout ! t'as juste trouvÃĐ une solution ÃĐvidente ;)

    • @nahil1721
      @nahil1721 āļ›āļĩāļ—āļĩāđˆāđāļĨāđ‰āļ§

      @@42ArthurDent42 dsns cette exo il devait juste trouver m et n donc sa marche en aucun cas c est ecris dÃĐmontrer

  • @PADABOUM
    @PADABOUM āļ›āļĩāļ—āļĩāđˆāđāļĨāđ‰āļ§

    Ma solution favorite, m=4 car 4080 est ÃĐgal à 255 x 2^4 et n est ÃĐgal 12 car 2^n > 4800 mais 2^(n-1)

  • @patricekoffi4474
    @patricekoffi4474 āļ›āļĩāļ—āļĩāđˆāđāļĨāđ‰āļ§ +1

    peux t-on utiliser le logarithme à base 2 ?

  • @Abi1one
    @Abi1one āļ›āļĩāļ—āļĩāđˆāđāļĨāđ‰āļ§

    TrÃĻs bien

  • @elyesayaida3325
    @elyesayaida3325 āļ›āļĩāļ—āļĩāđˆāđāļĨāđ‰āļ§ +12

    Moi ayant jouÃĐ au jeux 2048 je connais quasi toute les puissance de 2.
    J'ai compris cash que c'ÃĐtait 4096-16 plus qua trouve l'exposant.(2E12 2E4)

    • @blurp6206
      @blurp6206 āļ›āļĩāļ—āļĩāđˆāđāļĨāđ‰āļ§ +2

      pareil bahaha

    • @Laggron93
      @Laggron93 āļ›āļĩāļ—āļĩāđˆāđāļĨāđ‰āļ§ +4

      Attention, la notation "E" renvoie à l'ÃĐcriture scientifique! 2E12 veut dire 2 x 10^12.

    • @samueljehanno
      @samueljehanno āļ›āļĩāļ—āļĩāđˆāđāļĨāđ‰āļ§ +1

      Pareil mdr

    • @elyesayaida3325
      @elyesayaida3325 āļ›āļĩāļ—āļĩāđˆāđāļĨāđ‰āļ§ +1

      @@Laggron93 à oui mince.
      C'est juste que je voulais dire exposant.

    • @armand4226
      @armand4226 āļ›āļĩāļ—āļĩāđˆāđāļĨāđ‰āļ§

      Mais pourquoi vous en revenez tous à ces valeurs 4096 et 16 ?
      Que viennent elles faire ici ?

  • @romanjoulain7293
    @romanjoulain7293 āļ›āļĩāļ—āļĩāđˆāđāļĨāđ‰āļ§

    Pour ÃĐviter l'intuition sur la puissance de 2 à la fin:
    2^k-1 = 255
    2^k = 256
    kln2=ln(256)
    k=ln(2*2*2*2*2*2*2*2)/ln2
    k= (ln2+ln2+ln2+ln2+ln2+ln2+ln2+ln2)/ln2
    k = 8ln2/ln2
    k=8
    Au moins là un ordi peut le faire

  • @corazon540
    @corazon540 āļ›āļĩāļ—āļĩāđˆāđāļĨāđ‰āļ§

    meilleur prof ever c bon ?

  • @chronostone9320
    @chronostone9320 āļ›āļĩāļ—āļĩāđˆāđāļĨāđ‰āļ§ +4

    J'ai une autre mÃĐthode: on prend l'ÃĐcriture binaire de 4080 qui est 111111110000 il y 4 zÃĐros et 8 uns donc 2^4 et 2^12. Et on peut se convaincre que ça marche en essayant avec 2016

    • @happystoat99
      @happystoat99 āļŦāļĨāļēāļĒāđ€āļ”āļ·āļ­āļ™āļāđˆāļ­āļ™

      Bien vu! Codeur assembleur? :)

  • @PADABOUM
    @PADABOUM āļ›āļĩāļ—āļĩāđˆāđāļĨāđ‰āļ§

    Moi je suis toute de suite allÃĐ au racine de 4080 jusqu à trouver un nombre impair(255x4x4 ou 2^4) donc m-4 est ÃĐgal à 0 pour que 2^(m-4) soit ÃĐgal à 1
    AprÃĻs il reste jute le calcul pour n

  • @pureffm
    @pureffm āļ›āļĩāļ—āļĩāđˆāđāļĨāđ‰āļ§ +4

    J'ai fait une solution un peu plus intuitive sans passer par k:
    2^n - 2^m - 4080 = 0 2^4 ( 2^(n-4) - 2^(m-4) - 255) = 0 => 2^(n-4) - 2^(n-4) = 255
    Comme n> m , m = 4 (impossible d'obtenir un nombre impair sans m-4 = 0, seule une puissance entiÃĻre de 2 nulle peut crÃĐer un nombre impair)
    Donc 2^(n-4) - 1 = 255 2^(n-4) = 256 => n-4 = 8 < => n = 12

  • @bernardbrinette5388
    @bernardbrinette5388 2 āļŦāļĨāļēāļĒāđ€āļ”āļ·āļ­āļ™āļāđˆāļ­āļ™

    J'ai fait une approche un peu diffÃĐrente mais qui tient surement aussi la route. En fait, dans ce cas, 2^n sera toujours la puissance la plus proche juste au dessus du rÃĐsultat (4080). ça ne peut pas Être 2^n-1 car aucune autre puissance de 2 ne pourrait atteindre le rÃĐsultat, ni 2^n+1. Connaissant 1024 comme 2^10, je savais qu'effectivement 4096 ÃĐtait 4x1024 donc 2^(10+2) = 2^12. Donc n=12, puis la diffÃĐrence entre 4096 et 4080 ÃĐtant de 16, cela voulait dire m=4, puisque 2^4=16.

  • @Laggron93
    @Laggron93 āļ›āļĩāļ—āļĩāđˆāđāļĨāđ‰āļ§

    Je suis passÃĐ par des considÃĐrations bien pÃĐnibles pour montrer que 2^n - 2^m pour des entiers tels que n>m ÃĐtait compris entre 2^n-1 et 2^n pour prouver l'unicitÃĐ de n dans une telle ÃĐquation, pour ensuite dÃĐterminer m en connaissant n... Je me suis bien cassÃĐ la tÊte pour rien!

  • @quentin1982
    @quentin1982 āļ›āļĩāļ—āļĩāđˆāđāļĨāđ‰āļ§

    Difficile sans avoir la technique mais mathÃĐmatiquement c’est faisable pour beaucoup de monde

  • @stefancombski1921
    @stefancombski1921 āļ›āļĩāļ—āļĩāđˆāđāļĨāđ‰āļ§ +2

    Moi je savais que 2^12 = 4096. Du coup 4096 - 16 = 4080. Or 16 c'est 2^4. J'ai trouvÃĐ le rÃĐsultat en 15 secondes, mais il fallait connaÃŪtre les puissances de 2.

  • @mikaelderetour1933
    @mikaelderetour1933 āļ›āļĩāļ—āļĩāđˆāđāļĨāđ‰āļ§

    J’ai fait un raccourci parce que je connais les 1ere puissance de 2 , et 4080 trÃĻs proche de 4096. Puis 4096-4080 = 16, une autre puissance de 2â€Ķ. C’est un peu tricher mais ça marche. Pour des puissances de 3 c’ÃĐtait un autre problÃĻmeâ€Ķ

  • @mohamedkaram8094
    @mohamedkaram8094 āļ›āļĩāļ—āļĩāđˆāđāļĨāđ‰āļ§ +3

    J'ai essayÃĐ une methode plutot plus simple pour le resoudre
    On a : 2âŋ - 2áĩ = 4080
    J'ai essayÃĐ de lister les puissances de 2, et alors a noter que ( 2^12 = 4096 = 4080 + 16 ; et on sait que 2^4 = 16)
    On aura alors : 2âŋ = 4080 + 2áĩ
    On met : m = 4
    Ce qui donnera : 2âŋ = 4080 + 2^4 = 4096
    On dÃĐduit alors que : n = 12
    Sympa :D

    • @elc6927
      @elc6927 āļ›āļĩāļ—āļĩāđˆāđāļĨāđ‰āļ§ +2

      Salut j'aimerai savoir si possible tu es en quelle classe ? ( niveau d'etude ) Merci

    • @mohamedkaram8094
      @mohamedkaram8094 āļ›āļĩāļ—āļĩāđˆāđāļĨāđ‰āļ§ +1

      @@elc6927 j’ai deja fini mes etudes ( j’ai 25 ans 😁) je suis ingÃĐnieur

    • @elc6927
      @elc6927 āļ›āļĩāļ—āļĩāđˆāđāļĨāđ‰āļ§ +1

      @@mohamedkaram8094 super je te remercie

    • @mohamedkaram8094
      @mohamedkaram8094 āļ›āļĩāļ—āļĩāđˆāđāļĨāđ‰āļ§

      @@elc6927 t’as besoin de qlq chose ?

    • @elc6927
      @elc6927 āļ›āļĩāļ—āļĩāđˆāđāļĨāđ‰āļ§ +1

      @@mohamedkaram8094 Non en fait c'est juste que je suis en spÃĐ maths en 1ere et bon je m'en sors avec 14 de moyennes mais je suis super mauvais en maths ... Et ÃĐtant donnÃĐ que je veux faire maths expertes l'annÃĐe prochaine ( puisque je veux aller en PCSI) j'aimerai bien m'amÃĐliorer c'est tout. Quand j'ai vu ton commentaire je me suis demandÃĐ quel ÃĐtait ton niveau d'ÃĐtude puisque je n'ai su trouver la rÃĐponse...Voilà tout

  • @NDZ-wt7gb
    @NDZ-wt7gb āļ›āļĩāļ—āļĩāđˆāđāļĨāđ‰āļ§

    TrouvÃĐ toute seule 😊💊

  • @Ctrl_Alt_Sup
    @Ctrl_Alt_Sup āļ›āļĩāļ—āļĩāđˆāđāļĨāđ‰āļ§

    Quand on baigne dans l'informatique depuis l'enfance, on connaÃŪt les puissances de 2 au moins jusqu'à 8192.
    4080 ÃĐvoque de suite 4096 - 16

  • @ngagne7760
    @ngagne7760 5 āļŠāļąāđˆāļ§āđ‚āļĄāļ‡āļ—āļĩāđˆāļœāđˆāļēāļ™āļĄāļē

    En utilisant la fonction ln , on trouve n = 12 et m = 0 ou m = 12 et n = 0

  • @patricekoffi4474
    @patricekoffi4474 āļ›āļĩāļ—āļĩāđˆāđāļĨāđ‰āļ§ +2

    peux t-on utiliser le logarithme à base 2 pour le rapport 2^n/2^m = 2^(n-m) ?

    • @rudysoobaroyen3149
      @rudysoobaroyen3149 āļ›āļĩāļ—āļĩāđˆāđāļĨāđ‰āļ§

      C'est ce que j'ai egalement fait et je me suis avec une valeure de 12.99435 (5 pd) pour n et 11.99435 (5pd) pour m. Je pense que le prof n'a pas precise si les valeurs devaient etre naturelles ou reelles.

  • @Cangske
    @Cangske āļ›āļĩāļ—āļĩāđˆāđāļĨāđ‰āļ§ +5

    Bravo professeur 🙃🙃
    En tant qu'informaticien, on a l'œil pour les puissances de 2 (je vois dans les commentaires que beaucoup l'ont vu) et on trouve assez vite les solutions.
    Mais ta dÃĐmonstration est juste ÃĐnorme 😁😁
    Par contre, 4080 ou 2016 et pas d'autres, tu nous as pas dit pourquoi 🙃🙃

    • @leodew7846
      @leodew7846 āļ›āļĩāļ—āļĩāđˆāđāļĨāđ‰āļ§ +1

      je pense que sinon il y a plusieurs solutions possibles

    • @wildcat1139
      @wildcat1139 āļ›āļĩāļ—āļĩāđˆāđāļĨāđ‰āļ§ +1

      @@leodew7846 Non il n'y a toujours qu'une solution possible, et ça ne fonctionne que pour les nombres qui s'ÃĐcrivent en binaire 1...10...0

    • @leodew7846
      @leodew7846 āļ›āļĩāļ—āļĩāđˆāđāļĨāđ‰āļ§ +1

      @@wildcat1139 bah pour zero il y a une infinitÃĐ de solution

    • @wildcat1139
      @wildcat1139 āļ›āļĩāļ—āļĩāđˆāđāļĨāđ‰āļ§

      @@leodew7846 La dÃĐmonstration s'appuie sur n>m

  • @ladypacmon
    @ladypacmon āļ›āļĩāļ—āļĩāđˆāđāļĨāđ‰āļ§

    Pour moi ce fut une ÃĐvidence, je connais par coeur les puissances de 2 jusqu'à 2^13 xD

  • @confiance8143
    @confiance8143 10 āļŦāļĨāļēāļĒāđ€āļ”āļ·āļ­āļ™āļāđˆāļ­āļ™

    L'ÃĐcole c'est devenu trop facile à l'heure des nouvelles tech, rÃĐseaux sociaux. La seule contrainte c'est dÃĐsormais de s'assoir et travailler. âĪ

  • @esunisen3862
    @esunisen3862 āļ›āļĩāļ—āļĩāđˆāđāļĨāđ‰āļ§

    Je l'ai fait en genre 10 secondes sans rien dÃĐmontrer juste en connaissant les puissances de 2 par coeur ^^
    4080 = 4096 - 16

  • @AHMDZ696
    @AHMDZ696 āļ›āļĩāļ—āļĩāđˆāđāļĨāđ‰āļ§

    C'est beau, astucieux

  • @davidpasgoliath2673
    @davidpasgoliath2673 āļ›āļĩāļ—āļĩāđˆāđāļĨāđ‰āļ§

    en 3 sec dans ma tete : 4096 est une puissance de 2 ( en informatique on le sait ) du coup 4096 -4080 = 16 => 2m=16 => m=4 et 2n=4096 => n=12

  • @phildraw
    @phildraw āļ›āļĩāļ—āļĩāđˆāđāļĨāđ‰āļ§

    trop fort

  • @MR..mohamedelsayed
    @MR..mohamedelsayed āļ›āļĩāļ—āļĩāđˆāđāļĨāđ‰āļ§

    Is it possible to solve this equation [ 3 ^ m - 2^ m = 211 ] using the same steps as solving the equation [ 2 ^ m - 2^ n =4080 ] ?
    Assuming m is an integer .Of course, the solution, ( m = 5 ) how , is required to solve the steps

  • @leodew7846
    @leodew7846 āļ›āļĩāļ—āļĩāđˆāđāļĨāđ‰āļ§ +2

    vu la taille du nombre je me suit dit de la premiÃĻre puissance de 2 serait infÃĐrieur à 15 et la deuxiÃĻme beaucoup plus petite puis aprÃĻs juste chercher la puissance la plus proche donc 12 pour 4096 et donc 16 d'ÃĐcart soit 2^4

    • @maxilou8670
      @maxilou8670 āļ›āļĩāļ—āļĩāđˆāđāļĨāđ‰āļ§

      J'ai procÃĐdÃĐ de la mÊme maniÃĻre, beaucoup plus simple qu'une factorisation avec un changement de variable ... TrÃĻs bel exercice !

    • @jige1225
      @jige1225 3 āļŦāļĨāļēāļĒāđ€āļ”āļ·āļ­āļ™āļāđˆāļ­āļ™

      @@maxilou8670 OK, maintenant, faites pareil en base 3, 7, 13, 61, ...

  • @marcelnoguerol5040
    @marcelnoguerol5040 āļ›āļĩāļ—āļĩāđˆāđāļĨāđ‰āļ§

    Plus simple : 4096 -16 = 4080 donc (2puissance12 - 2puissance4) soit n=12 et m=4

  • @christophedidier6758
    @christophedidier6758 āļ›āļĩāļ—āļĩāđˆāđāļĨāđ‰āļ§

    Preums!
    2^12=4096, 2^4=16, 4096-16=4080 le compte est bon Laurent ! 😅 bon ayant fait un peu de programmation quand j’ÃĐtais jeune je me souvient de quelques puissances de 2 donc je l’ai vu tout de suiteâ€Ķ toutes les routes mÃĻnent à Rome !

    • @nid7819
      @nid7819 āļ›āļĩāļ—āļĩāđˆāđāļĨāđ‰āļ§

      Tu ne rÃĐponds pas à la question, tu montres juste que le couple (12,4) est solution de l’equation

    • @christophedidier6758
      @christophedidier6758 āļ›āļĩāļ—āļĩāđˆāđāļĨāđ‰āļ§

      @@nid7819 bahâ€Ķ si! D’aprÃĻs le thÃĐorÃĻme de l’homme fainÃĐantâ€Ķ si j’ai une solutionâ€Ķ bah c’est bon j’ai trouvÃĐ ! Non ?

  • @2stin
    @2stin āļ›āļĩāļ—āļĩāđˆāđāļĨāđ‰āļ§

    ça m'a sautÃĐ aux yeux que c'ÃĐtait 4096 - 16 donc direct 2^12 - 2^4

  • @quentinlardeau2867
    @quentinlardeau2867 2 āļŦāļĨāļēāļĒāđ€āļ”āļ·āļ­āļ™āļāđˆāļ­āļ™

    Perso j'ai rÃĐsonnÃĐ diffÃĐrent, j'ai remarquÃĐ que le nombre final ÃĐtait 0 donc que le dernier chiffre des rÃĐsultat des exposants sont les mÊmes et que en connaissant les puissances de 2 j'ai pue rapidement trouver n et m

  • @francefrance8659
    @francefrance8659 āļ›āļĩāļ—āļĩāđˆāđāļĨāđ‰āļ§

    Je ne suis pas entrÃĐ pour voir la solution, mais comment avez-vous ÃĐcrit un numÃĐro de puissance dans le titre

  • @jean-alexandrecamus8807
    @jean-alexandrecamus8807 āļ›āļĩāļ—āļĩāđˆāđāļĨāđ‰āļ§ +1

    Merci le jeu 2048 qui me rappelle a chaque fois les puissance de deux ... GrÃĒce à ça je sait que 4096 est ÃĐgal à 2^12 aprÃĻs le reste coule tout seul

    • @Christian_Martel
      @Christian_Martel āļ›āļĩāļ—āļĩāđˆāđāļĨāđ‰āļ§

      Exactement ce que je me suis dit. 😂 je suis arrivÃĐ rapidement à la rÃĐponse, car je trouvais que 4080 ÃĐtait plutÃīt proche de 4096. 😂

  • @eliseeorega6146
    @eliseeorega6146 11 āļŦāļĨāļēāļĒāđ€āļ”āļ·āļ­āļ™āļāđˆāļ­āļ™

    Pour trouver k on pouvait faire les ln aussi
    2exposant k -1=255
    2exposant k=254
    Kln2=ln254
    K=ln254/ln2
    K environ 7,9
    K =8
    😊

  • @GileadMaerlyn
    @GileadMaerlyn āļ›āļĩāļ—āļĩāđˆāđāļĨāđ‰āļ§ +7

    C'ÃĐtait vachement long comme mÃĐthode.
    Perso, ayant une formation d'informaticien, j'ai de suite vu que c'ÃĐtait 4096 - 16 donc 2^12 - 2^4, ça m'a pris 10 secondes xD

    • @paperyka8160
      @paperyka8160 āļ›āļĩāļ—āļĩāđˆāđāļĨāđ‰āļ§ +1

      Oui mais le but c'est de trouver toutes les solutions, donc en trouver une par intuition ne suffit pas en maths, il faut aussi montrer qu'il n'y en a pas d'autres !

    • @dlep9221
      @dlep9221 āļ›āļĩāļ—āļĩāđˆāđāļĨāđ‰āļ§

      😁 Idem, mÊme formation, j'ai donc vu tout de suite le 4096-16

    • @Srtnn...
      @Srtnn... āļ›āļĩāļ—āļĩāđˆāđāļĨāđ‰āļ§ +1

      @@paperyka8160 Tu as raison mais dans ce cas prÃĐcis il ÃĐtait demandÃĐ de dÃĐterminer LES entiers m et n qui conviennent. Cela sous-entend qu'un seul couple de valeurs convient. 👍

    • @herve6525
      @herve6525 āļ›āļĩāļ—āļĩāđˆāđāļĨāđ‰āļ§ +1

      @@Srtnn...l'ÃĐnoncÃĐ ne dit pas explicitement qu'il n'existe qu'un seul couple solution ;-)

    • @Srtnn...
      @Srtnn... āļ›āļĩāļ—āļĩāđˆāđāļĨāđ‰āļ§

      @@herve6525 Si si, c'est clairement sous-entendu. Tu fais des maths ? Si oui, alors tu sais que l'utilisation du "le" dÃĐcrit une unicitÃĐ. Si non, tu n'as pas ton mot à dire.

  • @z-ryk
    @z-ryk 9 āļŦāļĨāļēāļĒāđ€āļ”āļ·āļ­āļ™āļāđˆāļ­āļ™

    2^x - 2^y = 4080
    x > y , donc 2^y (2^(x-y) -1) = 4080
    4080 =
    4080 x 1
    2040 x 2
    1020 x 4
    510 x 8
    255 x 16
    au delà de 16, on est sur des nombres dÃĐcimaux.

    cas 2^y = 1:
    y = 0
    2^(x-y) -1 = 4080
    => 2^(x) = 4081
    => Impossible
    cas 2^y = 2:
    y = 1
    2^(x-y) -1 = 2040
    => 2^(x-1) = 2041
    => Impossible
    cas 2^y = 4:
    y = 2
    2^(x-y) -1 = 1020
    => 2^(x-2) = 1021
    => Impossible
    cas 2^y = 8:
    y = 3
    2^(x-y) -1 = 510
    => 2^(x-y) = 511
    => Impossible
    cas 2^y = 16:
    y = 4
    2^(x-y) - 1 = 255
    => 2^(x-4) = 256
    => x-4 = 8
    => x=12
    => (4,12)

  • @ronanb3
    @ronanb3 āļ›āļĩāļ—āļĩāđˆāđāļĨāđ‰āļ§ +2

    On a trouvÃĐ 2 entiers qui fonctionnent. Peut-on dÃĐmontrer que ce sont les seuls ?

    • @PADABOUM
      @PADABOUM āļ›āļĩāļ—āļĩāđˆāđāļĨāđ‰āļ§

      Oui, trÃĻs facilement. Cf mes autres commentaires

  • @rafaell-t4762
    @rafaell-t4762 āļ›āļĩāļ—āļĩāđˆāđāļĨāđ‰āļ§

    Je suis partis sur un autre raisonnement :
    On a 2^n -2^m = 4080, Or en dÃĐcomposant 4080 on trouve que 4080= 2^12 - 16 = 2^12 - 2^4. D’oÃđ : 2^n -2^m = 2^12 - 2^4.
    Donc à partir du moment oÃđ on part du principe que n et m n’ont qu’une solution possible alors n=12 et m=4.

  • @Shyynz
    @Shyynz āļ›āļĩāļ—āļĩāđˆāđāļĨāđ‰āļ§

    La mÃĐthode est ÃĐlÃĐgante mais ne faudrait-il pas montrer pour commencer que le couple {n,m} est unique ?

  • @thierryf2789
    @thierryf2789 7 āļŦāļĨāļēāļĒāđ€āļ”āļ·āļ­āļ™āļāđˆāļ­āļ™

    Pour moi: 2 m est plus petit que la moitiÃĐ de 2 n. 2 n = 4080+ 2 m et ne peut qu’Être ÃĐgal à 4096 donc n= 12 et m= 4

  • @meilleroland7813
    @meilleroland7813 āļ›āļĩāļ—āļĩāđˆāđāļĨāđ‰āļ§

    et si vous trouviez une solution à l'ÃĐquation X * X = pi

  • @alain9337
    @alain9337 āļ›āļĩāļ—āļĩāđˆāđāļĨāđ‰āļ§

    On cherche tout de suite une puissance de 2 >4080, on trouve immÃĐdiatement 4096
    Donc 4096-16=4080

  • @user-qi4di9ej5h
    @user-qi4di9ej5h āļ›āļĩāļ—āļĩāđˆāđāļĨāđ‰āļ§ +1

    il faut que vous deveniez prof

  • @mustaphaelmarkahi6511
    @mustaphaelmarkahi6511 āļ›āļĩāļ—āļĩāđˆāđāļĨāđ‰āļ§

    Si on connait les premiÃĻres puissances de 2...
    2^10=1024, 2^11=2048, 2^12=4096
    4096 = 4080 + 16 2^12 = 4080 + 2^4
    4080 = 2^12 - 2^4.

  • @Pima546
    @Pima546 āļ›āļĩāļ—āļĩāđˆāđāļĨāđ‰āļ§

    4080 J'ai tout de suite vu que c'ÃĐtait 4096 - 16 et du coup 2^12 - 2^4
    n=12
    m=4

  • @samyichalalen411
    @samyichalalen411 āļ›āļĩāļ—āļĩāđˆāđāļĨāđ‰āļ§ +2

    C olympiades collÃĻge ça nn ?

  • @befreedv6
    @befreedv6 āļ›āļĩāļ—āļĩāđˆāđāļĨāđ‰āļ§

    c'est de cette façon que j'ai trouvÃĐ aussi - du mÊme genre quel est le nombre dont la racine carrÃĐe et la racine cubique diffÃĻre de 18 ? avec un raisonnement analogue on peut trouver facilement

    • @corenthinoillic44
      @corenthinoillic44 āļ›āļĩāļ—āļĩāđˆāđāļĨāđ‰āļ§

      Ça revient à rÃĐsoudre x^3-x^2=18
      Du coup la rÃĐponse est 3

    • @quentind1924
      @quentind1924 āļ›āļĩāļ—āļĩāđˆāđāļĨāđ‰āļ§

      @@corenthinoillic44 Tu as mal compris la question. C’est la racine carrÃĐe et la racine cubique. Pas le carrÃĐ et le cube

  • @julientevuob4215
    @julientevuob4215 āļ›āļĩāļ—āļĩāđˆāđāļĨāđ‰āļ§

    En vrai j'ai trouvÃĐ uniquement car je connais les puissances de 2 pcq je bosse dans l'informatique.

  • @rekiaouhaji4776
    @rekiaouhaji4776 āļ›āļĩāļ—āļĩāđˆāđāļĨāđ‰āļ§

    اØŊŲ† اŲ„Ų…ØđاØŊŲ„ØĐ Ø§Ų„ØĩØŪŲŠØ­ØĐ Ų‡ŲŠ 2-2= 0 ŲˆŲ„اŲƒŲ† Ų„ŲƒŲŠ Ų†ØŪØĩŲ„ ØđŲ„Ų‰ 4080 ŲŠØŽØĻ ؊حØŊŲŠØŊ Ø­ØŽŲ… Ų†Ø§Ų†ŲˆŲ…ØŠØą Ųˆ Ų…ØŠØą

  • @risitasgg3465
    @risitasgg3465 āļ›āļĩāļ—āļĩāđˆāđāļĨāđ‰āļ§

    J'ai opÃĐrÃĐ par erreur et essaie j'ai trouvÃĐ n tels que 12 donc 4096 et m 4 pour 16 donc cela nous donne 4080

  • @GillesF31
    @GillesF31 āļ›āļĩāļ—āļĩāđˆāđāļĨāđ‰āļ§

    En informatique on sait que 4096 est 2^12 (sur la base du fondamental 2^10 = 1024) et comme 4096 - 4080 = 16 donc = 2^4. En consÃĐquence et en conclusion: 2^12 - 2^4 = 4096 - 16 = 4080. A propos, savez-vous pourquoi une pÃĒtisserie bien connue porte le nom de "mille-feuilles" ? Parce la pÃĒte, avant cuisson, est pliÃĐe 10 fois (ÃĐtirÃĐe et pliÃĐe 10 fois) selon la recette et que 2^10 = 1024 ... sauf qu'en cuisine on "arrondit" un peu (1024 devenant 1000). 🙂

    • @martin.68
      @martin.68 āļ›āļĩāļ—āļĩāđˆāđāļĨāđ‰āļ§

      Pour le mille-feuilles l'explication semble allÃĐchante mais comme souvent dans ces cas là personne ne va vÃĐrifier si c'est effectivement juste , sauf moi bien sÃŧr 😉
      Alors en rÃĐalitÃĐ il n'y a pas 10 pliages mais 6 dans la majoritÃĐ des cas, ensuite ce ne sont pas des pliages en deux.
      Mille est symbolique, c'est juste pour dÃĐsigner un grand nombre. Comme dans mille-pattes par exemple.

  • @Seyrox-oo4qi
    @Seyrox-oo4qi 7 āļŦāļĨāļēāļĒāđ€āļ”āļ·āļ­āļ™āļāđˆāļ­āļ™

    Je n'ai pas compris la partie de factorisation avant qu'on change par k

  • @nicolaschung916
    @nicolaschung916 āļ›āļĩāļ—āļĩāđˆāđāļĨāđ‰āļ§

    Etrange je trouve ça un peu simple pour un exo d'Olympiades, non ?

  • @VincentCama
    @VincentCama āļ›āļĩāļ—āļĩāđˆāđāļĨāđ‰āļ§

    RÃĐponse de chatGPT, c'est bien partie mais aprÃĻs ...
    Pour rÃĐsoudre cette ÃĐquation, il est utile de remarquer que 2^n - 2^m peut aussi s'ÃĐcrire comme 2^n - 2^m = 2^m (2^(n-m) - 1). Cette forme de l'ÃĐquation nous permet de voir que 2^(n-m) doit Être ÃĐgal à 41, car 2^(n-m) - 1 = 4080.
    Nous pouvons maintenant rÃĐsoudre cette ÃĐquation en utilisant la propriÃĐtÃĐ suivante : si a et b sont des entiers positifs tels que a^b = c, alors a = c^(1/b). En utilisant cette propriÃĐtÃĐ, nous pouvons ÃĐcrire 2^(n-m) = 41 comme 2^(n-m) = (41)^(1/(n-m)).
    Nous savons que 41 est un nombre premier, donc la seule façon pour qu'il soit ÃĐgal à une puissance de 2 est qu'il soit ÃĐgal à 2^5. Cela signifie que n-m doit Être ÃĐgal à 5. Nous ne savons pas encore la valeur de n et m, mais nous savons que n-m = 5.
    Nous pouvons maintenant rÃĐsoudre cette ÃĐquation en utilisant la propriÃĐtÃĐ suivante : si a et b sont des entiers positifs tels que a + b = c, alors a = c - b et b = c - a. En utilisant cette propriÃĐtÃĐ, nous pouvons ÃĐcrire n-m = 5 comme n = 5 + m et m = 5 + n.
    Nous pouvons maintenant utiliser ces ÃĐquations pour trouver les valeurs de n et m. Par exemple, si n = 5 + m, alors m = n - 5. Si n = 10, alors m = 5. Si n = 15, alors m = 10. Etc.
    En rÃĐsumÃĐ, pour que 2^n - 2^m = 4080, il faut que n-m = 5 et que n et m soient des entiers positifs. Plusieurs valeurs de n et m peuvent satisfaire ces conditions, par exemple (n,m) = (10,5), (15,10), (20,15), etc.

    • @VincentCama
      @VincentCama āļ›āļĩāļ—āļĩāđˆāđāļĨāđ‰āļ§

      En lui disant qu'il fait une erreur, c'est mieux, mais pas encore ça :
      Vous avez raison, 2^5 n'est pas ÃĐgal à 41. J'ai mal interprÃĐtÃĐ l'ÃĐquation 2^n - 2^m = 4080 dans ma derniÃĻre rÃĐponse en utilisant la propriÃĐtÃĐ qui permet de trouver la valeur de a lorsque nous connaissons la valeur de c et de b.
      Pour rÃĐsoudre l'ÃĐquation 2^n - 2^m = 4080, nous devons utiliser une autre approche. Une façon de rÃĐsoudre cette ÃĐquation est de remarquer que 2^n - 2^m = (2^m)(2^(n-m) - 1). Si nous connaissons la valeur de n-m, alors nous pouvons trouver la valeur de 2^(n-m) en divisant 4080 par 2^m.
      En utilisant cette approche, nous pouvons trouver que les valeurs de n et m qui satisfont l'ÃĐquation 2^n - 2^m = 4080 sont (n,m) = (12,4), (17,9), (22,14), etc. Si n = 12 et m = 4, alors 2^n - 2^m = 2^12 - 2^4 = 4096 - 16 = 4080, ce qui montre que ces valeurs de n et m satisfont bien l'ÃĐquation.
      Je suis dÃĐsolÃĐ pour les erreurs que j'ai commises dans mes rÃĐponses prÃĐcÃĐdentes. Si vous avez d'autres questions, n'hÃĐsitez pas à me les poser. Je ferai de mon mieux pour vous aider.

    • @VincentCama
      @VincentCama āļ›āļĩāļ—āļĩāđˆāđāļĨāđ‰āļ§

      et finalement :
      Vous avez raison, il n'y a que (n,m) = (12,4) comme solution de l'ÃĐquation 2^n - 2^m = 4080. Si n = 12 et m = 4, alors 2^n - 2^m = 2^12 - 2^4 = 4096 - 16 = 4080, ce qui montre que ces valeurs de n et m satisfont bien l'ÃĐquation.
      Dans ma derniÃĻre rÃĐponse, j'ai indiquÃĐ que les valeurs de n et m qui satisfont l'ÃĐquation 2^n - 2^m = 4080 sont (n,m) = (12,4), (17,9), (22,14), etc. Cette information ÃĐtait erronÃĐe, car seules les valeurs (n,m) = (12,4) satisfont l'ÃĐquation.
      Je suis dÃĐsolÃĐ pour cette erreur et pour toutes les erreurs que j'ai commises dans mes rÃĐponses prÃĐcÃĐdentes. Si vous avez d'autres questions, n'hÃĐsitez pas à me les poser. Je ferai de mon mieux pour vous aider.
      @hedacademy tu ne sera pas encore au chÃīmage !

    • @42ArthurDent42
      @42ArthurDent42 āļ›āļĩāļ—āļĩāđˆāđāļĨāđ‰āļ§

      @@VincentCama waouh, super test pour gpt4 je vais essayer ça sur d'autres trucs rigolos !

  • @balian9224
    @balian9224 āļ›āļĩāļ—āļĩāđˆāđāļĨāđ‰āļ§

    Alors moi j'ai ouvert Excel et listÃĐ les puissances de 2.
    J'ai vu que 2^12 soit 4096 ÃĐtait la premiÃĻre puissance de 2 supÃĐrieure à 4080 et il me restait 16 soit 2^4.
    Au delà de 12, aucune puissance de 2 minorÃĐe de 4080 ne donne une puissance de 2.

    • @michellepivert3964
      @michellepivert3964 āļ›āļĩāļ—āļĩāđˆāđāļĨāđ‰āļ§

      votre remarque est amusante , si la puissance x existe elle doit satisfaire l'ÃĐgalitÃĐ en dÃĐsignant par y l'ÃĐventuelle puissance de 2 espÃĐrÃĐe
      2px - 4080 = 2py soit 2px - 2py = 4080 GENIAL !

  • @caribou2073
    @caribou2073 āļ›āļĩāļ—āļĩāđˆāđāļĨāđ‰āļ§ +1

    Avis à tous les informaticiens : y'a pas que vous qui connaissez les puissances de 2...

    • @esunisen3862
      @esunisen3862 āļ›āļĩāļ—āļĩāđˆāđāļĨāđ‰āļ§

      Sans doute mais si ça avait ÃĐtÃĐ des puissances de 3 on aurait pas su xD

  • @AAArrakis
    @AAArrakis āļ›āļĩāļ—āļĩāđˆāđāļĨāđ‰āļ§

    Joooliâ€ĶâĪâĪâĪâĪ

  • @jean-francoisrouland1686
    @jean-francoisrouland1686 āļ›āļĩāļ—āļĩāđˆāđāļĨāđ‰āļ§ +2

    bonjour, moi aussi je suis passÃĐ par 4096 - 16; Est-ce que c'est valide comme raisonnement ?

    • @thierryf2789
      @thierryf2789 7 āļŦāļĨāļēāļĒāđ€āļ”āļ·āļ­āļ™āļāđˆāļ­āļ™

      Non, car il faut montrer que c’est la seule solution doti Être plus grand que 11 et plus petit que 13 car 2^m est I forcÃĐment plus petit que la moitiÃĐ de n

  • @rinkio9044
    @rinkio9044 āļ›āļĩāļ—āļĩāđˆāđāļĨāđ‰āļ§

    Tranquille...
    4080 = 4096 - 16
    n = 12
    m = 4

  • @mustaphakhezami7213
    @mustaphakhezami7213 āļ›āļĩāļ—āļĩāđˆāđāļĨāđ‰āļ§

    4080= 4096-16= 2^12 - 2^4 d'oÃđ n =12 et m= 4

  • @sidikikolacisse8789
    @sidikikolacisse8789 āļ›āļĩāļ—āļĩāđˆāđāļĨāđ‰āļ§

    A1aa